You are on page 1of 43

I dont think that there are more

challenging GMAT prep materials


on the market. Thank you! - Katia
DREAM
school!
Get
into your
Courses & Tutoring! info@manhattaneliteprep.com +1 888-662-6738
Comprehensive Review of GMAT Solution Strategy
M
A
N
H
A
T
T
A
N


E
L
I
T
E

P
R
E
P



G
M
A
T

E
L
I
T
E

S
T
R
A
T
E
G
Y

S
E
R
I
E
S


P
R
E
P
A
R
A
T
I
O
N

Q
U
I
C
K

G
U
I
D
E
THE LEADER
IN TEST PREP
FOR 10+ YEARS
GMAT

Elite Strategy Series
Preparation Quick Guide
Elite Strategy Series
MBA Quick Guide Preparation Quick Guide
Sentence Correction Guide Quantitative Quick Guide
Refer a friend & you each get $US50! www.manhattaneliteprep.com/students/
With the goal of helping the diverse array of high school,
college, graduate, and career seekers succeed, we launched
Manhattan Elite Prep. Our revolutionary education services
company offers elite test preparation courses, admissions
consultations, K-12 academic tutoring, language training and
career coaching! This new firm allows us to broaden our
footprints while guaranteeing the same committed, specialized
approach to candidates across all fields as we had done earlier
for a decade.
Our methods have been tested and proven to work over the past
decade and our success stories speak for themselves. We offer a
range of course options, making top GMAT preparation acces-
sible to busy schedules and various learning styles.
On behalf of our team, thanks for choosing Manhattan Elite Prep.
About Manhattan Elite Prep
We have been an industry leader in test prep and admissions
consultation services for over 10 years. Our team started as
Manhattan Review, focusing initially on GMAT preparation.
Working hard year in and year out, our efforts are marked by
substantial growth and success stories from former students
who have received that thick envelope letting them know
they've been accepted to Wharton, Harvard, Columbia, Yale, and
so many more!
Over time, many of our most valued customers came to our team
with concerns, such as how can your firm help me if I want to:
-sludy lor lhe GhL, 1OLlL, SA1, LSA1, or lhe MCA1
-prepare lor all high school lesls and K12 sub|ecls
-become lluenl in a loreign language
-improve my compuler or career skills such as cullural sensilivily
Testimonials
"I got my GMAT score up to 740. Thanks again for all your help. Hopefully Ill see you next year as a student."
1apiwa (London GMA1 crash course)
"Greal Course! Lxceeded my expeclalion. Kevin is knowledgeable and does a good |ob explaining lhe malerial."
Lmmanuel (Online GMA1 long course)
I completed a 10-hour tutoring session with Craig. And today, I am happy to report that my GMAT score has climbed up a total of 70 points
lo 680! Now l lall in lhe score range lop schools look lor."
-inmin (Online GMA1 luloring)
Americas-Europe-Asia-Online
anhattan
Admissions, Test Prep & Training
M ElitePrep
www.twitter.com/manhattanelitep
www.facebook.com/manhattaneliteprep
www.youtube.com/manhattaneliteprep
FREE RESOURCES
DISCOUNTS!
GMAT+ADMISSIONS VIDEOS
Free Admissions Consultations:
admissions@manhattaneliteprep.com
GMAT and GMAT CAT are registered trademarks of the Graduate Management Admission Council (GMAC).
GMAC does not endorse nor is it affiliated in any way with the owner of this product or any content herein.
anhattan
Admissions, Test Prep & Training
M
Elite
Prep







GMAT Verbal Concepts Guide



Elite Strategy Guide

Basic & Advanced Content

Comprehensive Analysis

Grammar Review

Accelerated Vocabulary

Sentence Correction Tips

Critical Reasoning Outline

Reading Comprehension
Strategies



www.ManhattanElitePrep.com
Copyright and Terms of Use

CopyRight and Trademark

All materials herein (including names, terms, trademarks, designs, images and graphics) are the
property of Manhattan Elite Prep, except where otherwise noted. Except as permitted herein, no
such material may be copied, reproduced, displayed or transmitted or otherwise used without the
prior written permission of Manhattan Elite Prep. You are permitted to use material herein for
your personal, non-commercial use, provided that you are not permitted to combine such material
into a combination, collection or compilation of material. If you have any questions regarding the
use of the material, please contact Manhattan Elite Prep at info@manhattaneliteprep.com

This material may make reference to countries and persons. The use of such references is for
hypothetical and demonstrative purposes only.

Terms of Use

By using this material, you acknowledge and agree to the terms of use contained herein.

No Warranties

This material is provided without warranty, either express or implied, including the implied
warranties of merchantability, of fitness for a particular purpose and non-infringement.
Manhattan Elite Prep does not warrant or make any representations regarding the use, accuracy or
results of the use of this material. This material may make reference to other source materials.
Manhattan Elite Prep is not responsible in any respect for the content of other source materials,
and disclaims all warranties and liabilities with respect to the other source materials.

Limitation on Liability

Manhattan Elite Prep shall not be responsible under any circumstances for any direct, indirect,
special, punitive or consequential damages (Damages) that may arise from the use of this
material. In addition, Manhattan Elite Prep does not guarantee the accuracy or completeness of its
course materials, which are provided as is with no warranty, express or implied. Manhattan
Elite Prep assumes no liability for any Damages from errors or omissions in the material, whether
arising in contract, tort or otherwise.

GMAT is a registered trademark of the Graduate Management Admission Council.
GMAC does not endorse nor is it affiliated in any way with the owner of this product or any
content herein.

10-Digit International Standard Book Number: 0-9824324-6-1
13-Digit International Standard Book Number: 978-0-9824324-6-4

Last Updated September 2012

http://www.manhattaneliteprep.com |info@manhattaneliteprep.com +1 (888) 662-6738
Free MBA Candidacy
Evaluation




Free MBA Candidacy Evaluation (via Phone or Email)
Manhattan Elite Preps Admissions Consulting staff will:
Get to know your strengths, personality, and goals
Help you market yourself to business schools effectively
Help you select & articulate the experiences most favorable to your application
Give you the competitive edge needed to gain admission into the most selective business schools


How does it work?
We offer free MBA Candidacy Evaluations over emails and on the phone to all students globally. You can
arrange a consultation via the following steps:
Step 1:
Email admissions@manhattaneliteprep.com to set up a consultation


Step 2:
Attach to the email
1. An updated resume
2. An outline of special circumstances that might be admissions impediments
3. A list of MBA program of choice by your own preference
4. A list of target application deadline and school year
5. A list of your strengths and weaknesses based on your self-evaluation


Step 3:
Up to three email exchanges or an equivalent of 20 minutes of phone conversation are included in this offer.
Let one of our consultant guide you through the admissions process!


http://www.manhattaneliteprep.com |info@manhattaneliteprep.com +1 (888) 662-6738

Course Coupons!
Your Discount Coupon Number is:
1210092022
10% off any Courses or Services
-OR-
1231437171
15% off any Combined Courses and Services
You will enter the coupon number on the payment page after you have registered for a course online.

- One student per coupon.
- Course coupons cannot be combined.
- The 10% coupon code is valid for each separate registration for one of
our courses or services; The 15% coupon code is valid for each
combined registration of any of our courses and services.
- Register and pay online 2 weeks before the course begins and receive
early registration discount and coupon discount!
- $200 Friend Referral program: new student receives $100 off their
course; referrer receives $100.

Your Refer-A-Friend Coupon Code:
1206666839
US$100 off A Full GMAT Course (16+ Hours)

Please enter the coupon number on the payment page after registering for a course online.
Please indicate the referrers or referees full name and course details.)



http://www.manhattaneliteprep.com | info@manhattaneliteprep.com +1-888-662-6738
Get into your
dream school!
Prepare your future
Once you have decided to pursue an MBA, you have essentially completed the first step in
the admissions process, and your motivation is a great precursor to success. But which
schools? What are the next steps? How do you demonstrate your strong drive and stand out
to an admissions board?
The admissions process overwhelms many applicants, especially busy, young business
professionals with limited time in their schedules. Our individually tailored guidance helps
you successfully traverse the pitfalls of the application process. From your application
essays to determining which school to attend, our admissions packages give you the expert
advice and knowledge necessary to make the most educated decisions possible.
Choosing to return to school is a big investment, but the value of an MBA from a top school
is indisputable. Our consulting services help you effectively present your experiences. and
create a roadmap to acceptance. Our consultants commit themselves to helping candidates
put forth their best application by building strong relationships with you and understanding
your nuances and gifts. They will help you stand out and look unique against the most
selective admissions standards.
Did You Know?
If you are a test prep student who signed up for a full course of 16 hours or
more you can join any available sessions of a comparable in-person course or
an online prep course within 60 days of the last day of your session.
Additionally, you will receive free access to our popular 100+ hours of
Online Recording Library.
anhattan
Admissions, Test Prep & Training
M ElitePrep

http://www.manhattaneliteprep.com |info@manhattaneliteprep.com +1 (888) 662-6738
Our Admissions Services
Essay and Resume Review
Includes the following service:
a) Grammar & Style Review;
b) In-depth Advice on the Theme & Coherency to Ensure a Compelling & Consistent Self-
Presentation
c) Up to Three Rounds of Exchanges between the Editor and the Student to Ensure Polished and Convincing Essays

Essay Review - Single Essay (< or about 800 words): US$150 (Global) ($50 for every additional 800 words in the same essay)
Essay Review - Mini Pack - Resume & 1 long (< or about 800 words) or 2 short (<250 words) Essays: US$200 (Global)
Essay Review - Basic Pack - Resume & All Essays per School: US$450 (Global)
Essay Review - Advanced Pack - Resume & All Essays for Two Schools: US$800 (Global)
Our Consultants will
Give you the competitive edge needed to gain admission into the most selective business schools
Help you select & articulate the experiences most favorable to your application
Get to know your strengths, personality, and goals
Help you market yourself to business schools effectively
Manhattan Elite Preps Admissions Consulting
staff is made up of former admissions committee
members and graduates from top Ivy League
schools. They have years of experience in
reviewing resumes, interviewing prospective
students, coaching on school choices, and
advising on how to best present yourself to
different schools.

We offer an array of application services from
comprehensive cover-to-cover Elite and
Dedicated Packages to goal-oriented a la carte
services such as mock interview preparation and
waitlist strategy. We can also prepare custom
admissions packages for applicants.
We offer the following services to applicants of college,
graduate schools, business schools, medical schools, law
schools and private high schools:

Elite Packages
Dedicated Packages
Essay Review
Application Strategy Consultation
Resume Review
Mock Interview
Waitlist Strategy
Case Interview
Rejection Letter Analysis
Scholarship Application Review
HBS 2+2 Dedicated Pack
Custom Packages

http://www.manhattaneliteprep.com |info@manhattaneliteprep.com +1 (888) 662-6738
Dedicated & Elite Packages
Elite Pack

1 School: $4000 2 Schools: $5000
3 Schools: $6000 4 Schools: $7000 5 Schools: $8000
Dedicated Pack

1 School: $1900 2 Schools: $2800
3 Schools: $3700 4 Schools: $4600 5 Schools: $5500

Assessment of Goals and Fit

Assessment of Goals and Fit

Application Strategy Formulation

Application Strategy Formulation

Crafting of Persuasive Essays

Crafting of Persuasive Essays

Construction of Powerful Resume

Construction of Powerful Resume

Mock Interview Preparation Sessions

Mock Interview Preparation Sessions

Solicitation of Effective Recommendations

Solicitation of Effective Recommendations

Advice on Scholarship/Loan Applications

Advice on Scholarship/Loan Applications

Waitlist/Deferral Strategy

Waitlist/Deferral Strategy

Re-application Strategy

Re-application Strategy

Interview question database

Prompt turnaround of materials 4-7 days

Sample essays, recommendations, resumes

7 day per week access to elite consultant

Online recordings related to your application

Prompt turnaround of materials 2-4 days

7 day per week access to elite consultant

Testimonials!
Our Elite and Dedicated packages give you the most complete,
end-to-end, comprehensive admissions consulting on the market.
"Thanks a lot for your thorough support throughout the application process! I just got
HBS offer. Without your help, I probably wouldnt have been able to turn around the
application package over the last 2-3 weeks while working full time." - Ying C.
"I was accepted to Columbia Business School for Early Decision Sept. 2011. With
Toms help, I was able to establish a strong message through my essays of why I
belong at Columbia. Derek K.


http://www.manhattaneliteprep.com |info@manhattaneliteprep.com +1 (888) 662-6738

GMAT
Preparation
Services
5 GMAT Score-Maximizing Strategies
Tip #1: Demonstrate to the CAT that
you can handle a variety of substantive
areas in all question formats to increase
your GMAT score. The greater the
variance among your ability in different
tested topics, the lower your score. In
other words, the GMAT rewards
generaliststest takers who demonstrate
a broad spectrum of competencies.
Tip #2: Try your best on the initial
questions. The first few questions you
answer will either move you to a
significantly more difficult or easy level;
however, the last few questions you
answer will only slightly increase or
decrease in difficulty.
Tip #3: Stay away from guessing at the
beginning of the test. However, if you
run out of time, then just randomly
answer the last questions, at least you
have 20% of the chance of answering each
question correctly. We need to caution
you against guessing in the early stage of
the test. Since your chances of guessing
correctly are only 20% for each question,
an incorrect choice moves you down to a
lower difficulty level very quickly at the
beginning of the test.
Tip #4: Please take particular care with
the first few questions of each question
type in both Verbal and Quantitative
sections. Sometimes, it might be well into
around the 10th question before you see a
new verbal type question. Whenever you
see a new type of question, proceed more
slowly and do your best without
unnecessarily spending too much time.
Tip #5: Practice makes perfect! Reduce
the response time in the areas where you
are best. That way you can save time for
the question types you are less confident
about. Take mock tests in a setting similar
to your test center during the same time of
the day.
Conclusion: Pace yourself! Taking the
GMAT is essentially a balancing act in
which you need to slow down and try
your very best at the beginning and pick
up the speed later till the end in order to
maximize your score.

Our Course Offers:
*Most Value-Added GMAT Course Pack*
1. Most Instruction Hours: Unlimited GMAT Class Access
2. Free Access to 100+ hours of GMAT Online Library
3. Our Better Score Guarantee: Unlimited Class Repeats and Advice
4. US$200 Student Referral
5. Student Discount to Private Tutoring & Admissions Help
6. Representative Instructor Bios

28 Classroom Hours 7-Session One-Week Intensive Courses
Course: One-week GMAT Intensive Course (Unlimited Class Access)
Regular: US$1650 (All payments within the last two weeks)
Promotional: US$1550 (Advance payments required!)

28 Classroom Hours 8- or 6-Session GMAT Long Course
Course: 6- or 8-week GMAT Long Course (Unlimited Class Access)
Regular: $1199 (All payments within the last two weeks)
Promotional: $1099 (Advance payments required!)

16 Classroom Hours 2-Session Weekend Crash Courses
Course: 2-day Weekend Crash Course (Unlimited Class Access)
Regular: $950 (All payments within the last two weeks)
Promotional: $850 (Advance payments required!)

----
GMAT Private Services (In-Person or Online)
Each package below requires a 2 hours minimum commitment and the
price is based on each sign-up.

Private Course - $1580: Dedicated 1-on-1 Sessions, Books, 5 CATs,
Online Access (Course and or Library, depending on availability)
Elite Pack (10+ Hrs) - $200/Hr: Priority; Books, Online Library, 5 CATs
Advanced Pack (10+ Hrs) - $115/Hr: Online Library, 5 CATs
Basic Pack (<10 hours) - $125/Hr

Private course combines the benefits of private tutoring with a structured
course topic outline. Students can learn at their own pace while focusing
on targeted concepts during each session.

Table of Contents

CHAPTER1-GMATELITESTRATEGYGUIDE 11
1.1 GMATOVERVIEWANDTESTFACTS 11
1.1.1 KEYTESTRELATEDRULES 2
1.1.2 TESTFEE 3
1.1.3 SCHEDULEYOURTEST 4
1.1.4 SCOREREPORT 4
1.1.5 SCOREDISTRIBUTION 5
1.2 GMATSECTIONSANDBASICCONCEPTS 7
1.2.1 GMATSECTIONS 7
1.2.2 BASICGMATCONCEPTS 9
1.3 KEYTEST-TAKINGANDPREPARATIONADVICE 11
1.3.1 GMATTESTTAKINGTIPS 11
1.3.2 GMATTESTPREPARATIONTIPS 12
1.3.3 GMATTESTDAYTIPS 13
1.4 KEYTESTSECTIONADVICE 14
1.4.1 PROBLEMSOLVINGTIPS 14
1.4.2 DATASUFFICIENCYTIPS 15
1.4.3 SENTENCECORRECTIONTIPS 16
1.4.4 CRITICALREASONINGTIPS 17
1.4.5 READINGCOMPREHENSIONTIPS 18
1.4.6 ANALYTICALWRITINGASSESSMENTTIPS 18
1.4.7 ABSOLUTEVS.PARTIALWORDS 19
1.5 FREQUENTLYASKEDQUESTIONS 22
1.5.1 TESTDAYFAQ 22
1.5.2 GENERALFAQ 26
1.5.3 COURSEFAQ 33


Chapter 1 - GMAT Elite Strategy Guide

1.1 GMAT Overview and Test Facts

Most of business school applicants must take the Graduate Management Admissions Test (GMAT). Beginning in
2009, many business schools started to accept the Graduate Record Exam (GRE) score as well.

The GMAT is a standardized test delivered in English. Unlike academic grades, which have varying significance based
on each schools grading guidelines, the GMAT scores are based on the same standard for all test takers and they help
business schools assess the qualification of an individual against a large pool of applicants with diverse personal and
professional backgrounds. The GMAT scores play a significant role in admissions decisions since they are
more recent than most academic transcripts of an applicant and they evaluate a persons verbal,
quantitative and writing skills.

The GMAT is a Computer Adaptive Test (CAT) and can be taken at any one of many test centers around the world 5
or 6 days a week. You may take the GMAT only once every 31 days and no more than five times within any 12-
month period. The retest policy applies even if you cancel your score within that time period. All of your scores and
cancellations within the last five years will be reported to the institutions you designate as score recipients.

The GMAT consists of four separately timed sections. The first section consists of an analytical writing task, also
known as Analytical Writing Assessment (AWA). The second section is known as the Integrated Reasoning section,
which was introduced in early June 2012, in replacement of a second essay originally in the AWA section. The
remaining two sections (Quantitative and Verbal) consist of multiple-choice questions delivered in a computer-
adaptive format. Questions in these sections are dynamically selected as you take the test to stay commensurate with
your ability level. Therefore, your test will be unique. Just one question is shown on the screen at a given time. It is
impossible to skip a question or go back to a prior question. Each problem needs to be answered before the next
question.

In both the Verbal and the Quantitative sections, everyone starts out with an average difficulty level. The difficulty of
subsequent questions then increases or decreases based on the correct or incorrect answers a person submits in the test.
For each correct answer you give, you are given a harder question for each subsequent question and for each incorrect
answer you are given an easier question. This process will continue until you finish the section, at which point the
computer will have an accurate assessment of your ability level in that subject area.

Your score is determined by three factors:
1) the number of questions you complete;
2) the number of questions you answer correctly and;
3) the level of difficulty and other statistical characteristics of each question.
To derive a final score, these questions are weighted based on their difficulty and other statistical properties, not their
position in the test.

Manhattan Elite Prep - Verbal Concepts Guide | GMAT Elite Strategy Guide

2
For the AWA section, one person and one computer programmed for grading (E-rater) score the essay based on essay
content, organization, grammar and syntactic variety. Your final, single score is an average of both individual cores
obtained on the essay. AWA scores are computed separately from other sections and have no effect on the Verbal,
Quantitative, or Total scores.

Although the GMAT score is considered as a reasonable indicator of future academic performance at business schools,
it does not measure your job performance, knowledge of business, interpersonal skills, and personality traits such as
motivation and creativity. Instead, your application, essays, recommendation letters and interviews will capture most
of those aspects.

1.1.1 Key Test Related Rules

General Management Admission Council (GMAC) is the owner of the GMAT and is in charge of setting the
standards for the exam itself including format, question types, difficulty levels, adaptive design, etc.

On January 4, 2006, Pearson VUE, not Educational Testing Services (ETS), began to administer the GMAT under a
new contract with the GMAC, which expires in 2013 and expects to be renewed.

We have summarized and prioritized the key rules below.

You can take the GMAT only ONCE every 31 days.

Though we generally recommend our students to ace the test on their first try, it is wise to leave yourself some
scheduling flexibility for a second attempt if necessary. Schedule your GMAT 5 to 6 weeks prior to your
application deadline.

A side note: If you receive a perfect score of 800, you may not retake the exam for 5 years.

Use of Erasable Laminated Graph Pad

The testing center will provide each candidate with a pad of 10 pages of yellow erasable laminated legal-size
graph paper and a special black-ink pen which resembles a fine point black-ink sharpie marker. Each page consists
of 33 rectangular boxes across and 71 down, with some margins around the border. Scrap paper is not used.

Page 1 displays a disclaimer and information on how to adjust your chair and pages 2 through 10 are yellow laminated
graph paper. The ink is erasable, but the testing center does not provide erasers, therefore if you do fill up the
whiteboard, the testing center will provide you with additional pages. Likewise, if the ink of your
marker starts to fade or the tip flattens, you may request a new one.

We think using graph paper is a good way to track the alphabetic choices given in a problem, sketch geometrical
figures to scale, and keep calculation steps in order. To get yourself familiar with the new instruments, try to
practice with laminated graph paper (or just graph paper or just laminated paper) and a sharpie style
pen.

Manhattan Elite Prep - Verbal Concepts Guide | GMAT Elite Strategy Guide

3
You cannot skip any section and must complete the entire test.

You must take the test in its set order and in its entirety, including the integrated reasoning and essay writing
section, or your scores will not be processed.

All scores and cancellations in the past 5 years will be on your score report.

All of the scores you received or cancelled in the last 5 years will be noted on your score report.

We recommend you only cancel your score if you are sure that your performance is not indicative of your
normal and true ability, due to unusual reasons such as health, emotions, accident, disturbing testing environment,
etc. By canceling the score, you avoid showing an inconsistency of your test performance, which might be a red flag
for admissions officers.

Otherwise, you should get your score so that you can get an objective evaluation of what you stand against other
GMAT test takers and your strengths and weaknesses. As long as you demonstrate consistent and improved test
results, reporting the score is generally preferred over cancellation.

You will receive your official score report online via an email notification 20 days after
test day. Paper score report will be available via mail upon request only.

Based on our students experience, it takes exactly 20 days for them to receive an email notification. You will still
receive an unofficial copy of your scores immediately after completing the exam and prior to leaving the testing center.
Typically you may fax or bring in a copy of the unofficial GMAT score report to be used to process your MBA
application until the official scores arrive from the testing services. MBA programs usually can use the unofficial score
report to make a recommendation on an application, but the official GMAT scores must reach the school
before an official offer of admission can be made.

1.1.2 Test Fee

The fee to take the GMAT is U.S. $250 worldwide. The fee for rescheduling the date, time, or location of the test is
U.S. $50 for each appointment you change.

When you want to reschedule the GMAT, to avoid the forfeiture of your test fee, you must allow at least 7 calendar
days between the day you reschedule your appointment and your test day. Appointments cannot be rescheduled for a
date that is more than one year after the original appointment date.

When you cancel the test appointment, a partial refund of U.S. $80 will be given if it is canceled at least 7 calendar
days before your original test day.

The rescheduling fee and cancellation refund amount are subject to change without notice.

Manhattan Elite Prep - Verbal Concepts Guide | GMAT Elite Strategy Guide

4


1.1.3 Schedule Your Test

When setting a test date and look up test centers at http://www.mba.com/mba/TaketheGMAT, keep in mind the
following:

Consider the times of day you tend to be able to concentrate best. Take your test in the morning or afternoon
accordingly.

Make sure the week before your test day will not be a stressful one. This will help you to concentrate, be well rested,
calm and in the right frame of mind to ace the GMAT.

Be aware of application deadlines and do your best to provide yourself with enough time after the exam to focus on the
other parts of your Business School applications.

Remember to select:

The best possible time of day for you.

A low stress week.

AND give yourself sufficient time to prepare fully for the test.

In the final week before your test, remember:

Dont cram.

Take a practice test.

Get solid rest.

1.1.4 Score Report

Your Total score and Quantitative and Verbal section scores are available upon your completion of the test. The only
opportunity that you will have to cancel your scores is immediately (within 2 minutes) after you complete the test, but
before you view your scores. A message will ask you if you want to cancel your scores. You cannot cancel your
scores after they are displayed or reported to you.

If you cancel your scores, they cannot be reinstated later. A score cancellation notice will be sent to you and your
selected schools. It will remain a part of your permanent record and will be reported on all of your future score reports.
The test will not be refunded and will be accounted for as one taken test.

The official score report is available online instead of being mailed to you. Through a direct email 20 days after the
test, you will be notified of the accessibility of your online official score report which is also available to the schools
you selected as recipients. Official Score Reports are now mailed to the student by request only.

Manhattan Elite Prep - Verbal Concepts Guide | GMAT Elite Strategy Guide

5
Official GMAT score reports, which include the AWA score and Integrated Reasoning score, will be mailed to you
and your designated score report recipients (schools) approximately two weeks after the test. You must respond to the
essay, the integrated reasoning questions, and each multiple-choice section of the test to get an official score report.

During the test, if you click ``Section Exit'' or ``Test Quit,'' you will have to confirm your choice. If you clicked it by
mistake or change your mind, just select the option ``Return to Where I Was.'' Once you exit a section or quit a test,
you wont be able to return to it and wont receive a score for any section, regardless how many questions you have
answered.

You may take the GMAT only once every 31 days and no more than five times within any 12-month period. The
retest policy applies even if you cancel your score or quit a test within that time period. Official GMAT score results
are kept on file for 10 years. All your scores and cancellations within the last five years will be reported to the
institutions you designate as score recipients.

On your test day, you may select up to five schools to receive your scores before you take the test. Once you have
made your selection, you will not be able to change or delete the list of schools. If you would like to send your scores
to more schools, you may order additional score reports at a cost of U.S. $28 per school.

You may request that your essay be rescored if you have reason to believe that your AWA scores are not accurate. The
multiple-choice quantitative and verbal sections of the test cannot be rescored. Independent readers will rescore your
essay for a fee of U.S. $45.

Requests for rescoring must be made within six months of your test date. Rescoring may result in increases or
decreases in your original AWA score. The rescoring results are final. Revised results will be sent to you and the
schools you designated as score recipients within three weeks of your request.

1.1.5 Score Distribution

Total GMAT scores range from 200 to 800. About 62 % of test takers score under 600. The Verbal and Quantitative
scores range from 0 to 60. For the Verbal section, most people score between 9 and 44. For the Quantitative section,
common scores are between 7 and 50. The Verbal and Quantitative scores measure different things and cannot be
compared to each other, however, each sections score can be compared across different GMAT tests.

Your GMAT score is an important part of your overall application.

If you receive a score below 500, we recommend that you retake the exam. A score below 500 will make
acceptance to any school rather difficult.

A score below 600 will make acceptance into a top school unlikely without an otherwise flawless application.

A score in the range of 600-700 will help keep you in the running for acceptance into a top business school.

A score above 700 is terrific and will help improve your MBA applications.

Scaled scores of 760 out of 800 on the combined test generally correspond to the 99
th
percentile.

Manhattan Elite Prep - Verbal Concepts Guide | GMAT Elite Strategy Guide

6
GMAT Test Scores Distribution Snapshot - Total Score




Scaled scores of 760 out of 800 on the combined test generally correspond to the 99
th
percentile.

680 out of 800 corresponds to the 85
th
percentile.

GMAT Test Scores Distribution Snapshot - AWA Score


0%
44%
62%
78%
85%
90%
99%
99%
0% 20% 40% 60% 80% 100%
200
540
600
650
680
700
760
800
GMATTotalScoreDistribution
PercentileScore
0%
6%
20%
39%
57%
77%
90%
0% 20% 40% 60% 80% 100%
0
3
4
4.5
5
5.5
6
GMATAWAScoreDistribution
PercentileScore
Manhattan Elite Prep - Verbal Concepts Guide | GMAT Elite Strategy Guide

7
One person and one computer programmed for grading (E-rater) score each AWA based on essay content,
organization, grammar and syntactic variety. E-rater is an electronic system that evaluates more than 50 structural and
linguistic features. College and university faculty members trained as reviewers of the AWA essay consider the overall
quality of your ideas, your overall ability to organize, develop, and express those ideas, the relevant supporting reasons
and examples you cited, and your ability to write in standard written English. In considering the elements of standard
written English, reviewers are trained to be sensitive and fair in evaluating the essays of non-native English speakers.

E-rater and independent readers agree, on average, 87 % to 94 % of the time. If the two ratings differ by more than one
point, another evaluation by an expert reader is required to resolve the discrepancy and determine the final score.

Graders assign scores out of 6.0 based on intervals of 0.5 points. The AWA score is computed separately and have no
bearings on any other GMAT scores.

1.2 GMAT Sections and Basic Concepts

1.2.1 GMAT Sections

The GMAT includes the following sections:

Quantitative Section Verbal Section Essay / Integration Reasoning Section

Problem Solving Sentence Correction Analytical Writing Assessment
(AWA)
Data Sufficiency Critical Reasoning

Integrated Reasoning (IR)
Reading Comprehension

Each section requires its own specific strategy, but you may apply some techniques to all sections.
Manhattan Elite Prep - Verbal Concepts Guide | GMAT Elite Strategy Guide

8

Section # of
Quest.
Time
Allowed
Details Score
Range
Score
Interval
Official Score
Report (within
20 days of test
day)
Unofficial Score
Report (right
after the test)
Essay 1 30 min Analysis of an
Argument (30 min., 1
topic)
0 6 0.5 Yes No; Scored
separately
Integrated
Reasoning
12 30 min Multi-Source
Reasoning
Table Analysis
Graphics Interpretation
Two-Part Analysis

1 - 8 1 Yes No; Scored
separately
Optional 8 minutes
Break

Quantitative 37 75 min Problem Solving (23-24
questions)
Data Sufficiency (13-14
questions)

0 - 60 1; scores <7
or >50 are
very rare
Yes Yes
Optional 8 minutes
Break

Verbal 41 75 min Critical Reasoning (14-
15 questions)
Sentence Correction
(14-15 quest.)
Reading
Comprehension
(4 passages, 12-14
questions)

0 - 60 1; scores <9
or >44 are
very rare
Yes Yes
Total 1 AWA;
12 IR;
78
Multiple-
choices
4 hours
(approx.)
- 200-800 10; scaled
from Verbal
and
Quantitative
sub-scores
Yes Yes

Please note that not all of the verbal and quantitative questions are scored. In the Verbal section, approximately 37 of
the 41 questions are scored, and in the quantitative section, approximately 33 of the 37 questions are scored. The un-
scored questions are there for the purpose of gauging results for future tests. Within the quantitative section, the
different types of math questions are intermixed.

An Analysis of an Argument essay will appear in the AWA section.

Manhattan Elite Prep - Verbal Concepts Guide | GMAT Elite Strategy Guide

9
1.2.2 Basic GMAT Concepts

Various GMAT sections test students understanding of fundamental quantitative concepts, and their knowledge, skills
and analytical ability. To perform well on the test, students must master the basic underlying math and grammar
principles and typical question types. The following is a quick overview.

Quantitative Section

The Quantitative section measures your basic mathematical skills, understanding of elementary concepts, and the
ability to reason quantitatively, solve quantitative problems, and interpret graphic data. Problem-Solving and Data-
Sufficiency questions are intermingled throughout the section.

Basic Concepts

Integers and Prime numbers

Fractions and Percentages

Mark-up and Margin

Exponents and Roots

Equations and Inequalities

Probability, Permutations and Combinations

Statistics, Graph and Data Interpretation

Coordinate Geometry, Area and Volume of Various Geometrical Objects

Others to Be Discussed in Class

Major Question Types

Solving Equations

Profit, Cost and Break-Even Calculations

Distance-Rate-Time Problems

Divisibility

Averages and Weighted Averages

Word Problems

Data and Graph Interpretation

Area and Volume of Geometrical Shapes
Manhattan Elite Prep - Verbal Concepts Guide | GMAT Elite Strategy Guide

10

Mixture Problems

Others to Be Discussed in Class

Verbal Section

The Verbal section of the GMAT measures your ability to:

Correct written English to conform to standard grammar rules and styles

Read, reason and evaluate arguments

Speed read, comprehend and assess written English articles

Basic Grammar Rules:

Subject-Verb Agreement

Pronoun Reference

Prepositions and Articles

Verb Tenses and Voices

Parallelism

Idiomatic Usages

Others to Be Discussed in Class

Basic Question Types:

Please review the following introductory sections

Manhattan Elite Prep - Verbal Concepts Guide | GMAT Elite Strategy Guide

11
1.3 Key Test-taking and Preparation Advice

The scores necessary to get into top schools are increasing year by year, making quality preparation an even greater
necessity. Disciplined and dedicated preparation for the GMAT will allow you to get the best score possible on the
exam and get into the school of your choice.

High quality preparation is essential to achieving your best score on the GMAT. High quality preparation means
becoming intimately acquainted with the test structure, format, and the types of questions that are being asked. It
means improving upon your weak areas through practice and repetition. It means developing your ability to answer
correctly the tougher questions. It also means becoming aware of the types of answers that tend to be the correct ones.

In terms of general GMAT taking strategies, we recommend:

Learn the most typical problems and answer types.

Answer all questions.

Guess and estimate when necessary.

Try your best on the initial questions.

Others to be discussed in class.

1.3.1 GMAT Test Taking Tips

If you aim to achieve a top score, we recommend these Five GMAT CAT Score-Maximizing Strategies:
Tip #1: Demonstrating to the CAT that you can handle a variety of substantive areas in all question
formats will increase your GMAT score.

The greater the variance among your ability in different tested topics, the lower your score. In other words, the
GMAT rewards generaliststest takers who demonstrate a broad spectrum of competencies. This approach does
make sense in a business world where being well-rounded and knowledgeable is often positively correlated to a
managers decision-making skills and managerial ability.

Tip #2: Try your best on the initial questions.

The first few questions you answer will either move you to a significantly more difficult or easy level; however,
the last few questions you answer will only slightly increase or decrease in difficulty.

Questions at the beginning of the test and first question in each question type are of moderate difficulty. If you
answer this question correctly, then the difficulty level increases.

Tip #3: Take particular care with the first few questions of EACH QUESTION TYPE in both Verbal and
Quantitative sections.

Sometimes, it might be well into around the 10th question before you see a new verbal type question. Whenever
Manhattan Elite Prep - Verbal Concepts Guide | GMAT Elite Strategy Guide

12
you see a new type of question, proceed more slowly and do your best without unnecessarily spending too much
time.

Tip #4: Take all questions seriously and answer all of them.

Use a systematic approach to your test. Skipping questions is not helpful on the GMAT CAT.

Stay away from guessing at the beginning of the test. However, if you run out of time, then just randomly answer
the last questions, at least you have 20% of the chance of answering each question correctly.

We need to caution you against guessing in the early stage of the test. Since your chances of guessing correctly
are only 20% for each question, an incorrect choice moves you down to a lower difficulty level very quickly at
the beginning of the test. Please also bear in mind that there is a penalty for not finishing a section. For each
unfinished section, the penalty is about 4x the point for an incorrect answer.

Tips #5: Maintain strong momentum from beginning to end as the final problems can sometimes be equally
as important as the initial ones.

Tip #6: Reduce the response time in the areas you are best at.

For example, try cutting your Sentence Correction time to less than one minute per question. That way you can
save time for the question types you are less confident about.

Conclusion: Pace yourself! Taking the GMAT is essentially a balancing act in which you need to slow down and try
your very best at the beginning and pick up the speed later till the end in order to maximize your score.

1.3.2 GMAT Test Preparation Tips

Practice makes perfect!

During your GMAT preparation, be sure to incorporate the following study skills:

Simulate the real test setting as much as possible.

Take mock tests in a setting similar to your test center during the same time of the day.

Create a study environment that is as similar as possible to the actual testing setting, which typically includes a
quiet space, possibly a computer room or office environment, or set up a serious test-taking environment in your
house or a public library or a park or another facility so that you can exclusively focus on taking the mock tests.

Do not take breaks during a practice test. In the actual test center, you will not be allowed coffee breaks etc.
during sections.

Eliminate distractions and be conscious of time. Especially when you taking practice tests, be as aware of the
clock as you will need to be on the actual exam.

Remember that the actual exam is on the computer so take advantage of opportunities to practice with Computer
Adapted Tests. For many test-takers, reading large amounts of material on the screen is not easy. It not only dries
out their eyes but also makes it hard to absorb the material. Simply practice reading etc. on the computer. The
only way to improve is to practice. So read long articles on your computer screen.
Manhattan Elite Prep - Verbal Concepts Guide | GMAT Elite Strategy Guide

13

Make a special effort to improve your weaknesses along with strengthening your expertise during practice.

After completing a practice test, be sure to go over the questions you answered incorrectly.

This is the only way to improve. You must understand your mistakes so that you will not make them on the test.
(Manhattan Elite Prep provides you with over 100 pages of detailed solution guides!)

Prepare with as many CATs as possible from all sources.

Save the free GMATPrep software from www.mba.com for right before the actual test and practice the
tests in a mock test setting of your choice so that your memory of the actual question types and difficulty
levels which have appeared in prior GMAT tests stays fresh.

Make a list of those typical errors you tend to make and consciously remind yourself of them and refrain
from making the same mistakes.

The day on or before your actual test, review all prior mistakes along with explanations.

Keep a light-hearted and positive attitude on the test day.

1.3.3 GMAT Test Day Tips

Here is a summary of to-do items for the test day:

Bring all necessary documents such as identification cards (I.D.), the registration ticket and the names of the
schools to which you would like send your test score. (Bring two forms of ID in case one has expired or is not
acceptable to the test center.)

Bring something warm to put on in case the room is too cold.

According to test center rules, you have to wear the sweater or coat instead of just putting it around your shoulders.
So make sure that the additional layer of sweater or coat is comfortable in a test-taking setting.

Also be sure you can remove a layer of clothing in case the room is hot.

Bring something light to drink or eat.

A bottle of water or a soft drink with a cap is preferred over a can so that you can minimize the chances of spilling.
(Though you can not take anything into the testing room, you will be assigned a small locker. During your 5-
minute breaks, you can have a few sips to stay hydrated or a bite to eat if you get hungry. Normally test centers
allow you to put it outside on a desk or at an easily reachable spot or inside your locker so that you can quickly
grab the drink or the food.)

No testing aids such as study notes, calculators and PDAs are allowed.

At the test:

Follow your normal routine.

Manhattan Elite Prep - Verbal Concepts Guide | GMAT Elite Strategy Guide

14
Arrive at the test at least 30 minutes early.

Do concentrate on the first 10 questions of each section most. At the beginning of each section, the total number of
questions and the total time allowed are stated.

Maintain a focused mind and a positive winning attitude throughout the entire test.

Do not panic. Focus on one question at a time. Focus on one section at a time. Do not think beyond your current
section and lose your concentration.

Do not get fixated and spend unreasonable time on any single question. It will not make or break your score.
Because the score per section is partially based on the number of questions you answer, try to answer as many
questions as you can.

Do not leave any questions unanswered before the section time runs out. Always submit an answer after some
educated or blind guesses. Remember that you cannot skip questions or change an answer once you confirm it.

If a few questions or passages are difficult to understand, do not let them prompt you to cancel your score entirely.
You never know.

If something is wrong with the computer, or if someone is bothering you, or if its miserably hot etc., signal to an
exam proctor. The proctor walks around in the test room every 15-20 minutes.

Pace yourself and keep track of your progress by checking the amount of time you have left on the test screen.
Each section is 75 minutes. You have about two minutes per Quantitative question and about 1.75 minutes per
Verbal question.

Pay attention to the number of questions that remain in a section. There are 37 quantitative section questions.
There are 41 verbal section questions.

Clicking on ``HELP'' or hiding the ``TIME'' information doesnt pause or stop the time.

Between test sections, ask for new supply of laminated scratch pads if necessary.

Take advantage of breaks. Rest your eyes, as the computer screen is difficult to stare at for 4 hours straight.

1.4 Key Test Section Advice

1.4.1 Problem Solving Tips

The Problem Solving section of the GMAT tests your ability to solve questions and derive correct answers. Often these
problems present you with an algebraic formula. It is important that you develop a good pace in your preparation for
this section, as speed together with precision will help you to do your best on the GMAT exam.

Main Strategies:

First identify the underlying key mathematical concept of the problem.
Manhattan Elite Prep - Verbal Concepts Guide | GMAT Elite Strategy Guide

15

Determine the best way to approach the problem at hand. Common strategies include:

Applying algebraic and geometric formulae
Back solving
Approximation
Elimination

Check your work and read the question again. You may have solved the problem correctly but simply failed to
answer the question being asked.

Take educated guesses when you see fit to do so.

Others to be discussed in class.

Key test preparation principles include:

Maintain speed and precision.

Remember your Algebra and Geometry concepts.

Review your Algebra and Geometry definitions.

1.4.2 Data Sufficiency Tips

Data-Sufficiency questions are designed to measure your ability to:

Analyze a quantitative problem.

Recognize relevant information.

Determine whether there is sufficient information to solve a problem.

Data-Sufficiency questions are accompanied by some initial information and two statements, labeled (1) and (2). You
must decide whether the statements given offer enough data to enable you to answer the question.

Data Sufficiency questions do not ask for actual number solutions, and instead they ask simply: Is the
information given adequate to solve a question?

Two statements are laid out as two possible conditions. It is important to analyze each statement independently from
the other statement. In other words, you cannot mix the information from one statement with the other.

There are two common types of Data Sufficiency questions:

Close-ended: Is Y divisible by 3?

Open-ended: What is the value of X?

There are five answer choices for Data Sufficiency.

Manhattan Elite Prep - Verbal Concepts Guide | GMAT Elite Strategy Guide

16
A: Statement (1) is sufficient and (2) is insufficient.
B: Statement (1) is insufficient and (2) is sufficient.
C: A combination of both statements is sufficient. Either statement alone is insufficient.
D: Both statements are sufficient independently.
E: Neither the combination nor either individual statement is sufficient.

In a close-ended question, you can judge whether each statement is sufficient by determining if its answer is always
Yes or always No. A statement is insufficient if its answer is sometimes Yes or sometimes No.

In an open-ended question, you can judge whether each statement is sufficient by determining if its answer results in a
single value. A statement is insufficient if its answer leads to a range of values, instead of a specific value.

Other strategies include:

Memorize the standardized answer choices for Data Sufficiency questions.

Evaluate each statement or equation individually and then together.

Others to be discussed in class.

1.4.3 Sentence Correction Tips

The Sentence Correction section tests your knowledge of written English grammar by asking you which of the five
choices best expresses an idea or relationship. This section gives you a sentence that may or may not contain errors of
grammar or usage. You must select either the answer that best corrects the sentence or the answer stating that the
sentence is correct as is. The questions will require you to be familiar with the stylistic conventions and grammatical
rules of standard written English and to demonstrate your ability to improve incorrect or ineffective expressions.

This section tests two broad aspects of language proficiency:

Correct expression

Effective expression

A correct sentence is grammatically correct and structurally sound. It conforms to all the rules of standard written
English such as subject-verb agreement, verb tense consistency, modifier reference and position, idiomatic expressions
and parallel construction.

In addition to being correct, a sentence needs to be effective. It should express an idea or relationship clearly and
concisely, as well as grammatically. A best choice should have no superfluous words or unnecessarily complicated
expressions. This does not mean that the shortest choice is always the best answer. Proper diction is another important
part of effectiveness. It refers to the standard dictionary meanings of words and the appropriateness of words in
context. In evaluating the diction of a sentence, you must be able to recognize whether the words are well chosen,
accurate, and suitable for the context.

One common error that test takers often make in the Sentence Correction section is choosing an answer that
sounds good. Do not go on with your gut feeling in this section. Remember your grammar and look for errors in
construction (e.g., noun-verb agreement) and eliminate answers that you are sure are incorrect.

Look over answer choices and note for clearly identifiable patterns.
Manhattan Elite Prep - Verbal Concepts Guide | GMAT Elite Strategy Guide

17

Focus your attention on the most crucial grammatical issue in the problem.

Look for patterns in the first and last word(s) across all answer choices, as those parts of the sentence often
demonstrate major grammatical differences.

Look over each answer choice, as you can eliminate the wrong answer by focusing on not only the grammatical
difference from other choices, but also the answers own language style, word usage and idiomatic expressions.

Keep in mind also that the GMAT prefers active to passive constructions.

Eliminate clearly incorrect answers.

Select the answer which is grammatically correct, idiomatically acceptable and the most effective.

Others to be discussed in class.

1.4.4 Critical Reasoning Tips

The Critical Reasoning section tests your ability to make arguments, evaluate arguments, and formulate or evaluate a
plan of action. In this section we recommend that you read the questions carefully and identify the assumption implicit
in the statement with a heightened awareness of any weakness in the argument.

Please bear in mind the following when you practice in this section:

Break the argument down into its parts: conclusion, evidence and assumption.

The main assumption types include:

Cause and Effect
Comparison/Analogy
Representative Sample
Number and Logic Based
Implementation

Be familiar with major critical reasoning questions category:

a. Assumption Questions
Assumption
Weakening
Strengthening
Flaw
b. Inference/Conclusion Questions
c. Paradox/Explain Questions
d. Method of Reasoning Questions

Determine how the question fits into these types.

Eliminate clearly incorrect answers when they are totally irrelevant or the opposite of the desired answer.

Manhattan Elite Prep - Verbal Concepts Guide | GMAT Elite Strategy Guide

18
Others to be discussed in class.

1.4.5 Reading Comprehension Tips

You may see as many as 4 passages in the Reading Comprehension section up to 350 words in each passage, followed
by 3 or 4 interpretive, applied, and inferential questions. The topics are typically related to social sciences such as
politics and history, physical or biological sciences such as geology and astronomy, business-related areas such as
marketing, economics and human resource management, along with other advanced subjects.

Because the Reading Comprehension section includes passages from various different content areas, you may have
general knowledge about some of the topics. However, no specific familiarity of the material is required. All questions
are to be answered on the basis of what is stated or implied in the reading material.

Reading Comprehension questions measure your ability to speed read, understand, analyze, and apply information and
concepts presented in written English. It evaluates your ability to:

Understand words, terms and statements.

Understand the ideas, concepts and logical relationships between significant perspectives and to evaluate the
importance of arguments.

Draw inferences from facts and statements.

Understand and follow the development of quantitative concepts as presented. Then interpret and use the data to
reach conclusions.

We recommend the following general guidelines:

Quickly scan the passage to understand the underlying theme. Read the first and last sentence particularly
carefully.

Read the passage in detail. Note its main structure.

Pay special attention to the usage of transitional words that change the passages flow, such as yet, though,
however, despite, etc. These transitional words often highlight important information in the passage.

Note different perspectives presented and the relationship amongst them.

Quickly scan through the questions and answers to develop a general sense of the focus of the question.

Read the answers to note the possibilities addressed. At this point eliminate any clearly wrong answers.

Others to be discussed in class.

1.4.6 Analytical Writing Assessment Tips

The first section you will encounter on the GMAT is the Analytical Writing Assessment (AWA). The AWA requires
you to write 1 essay within 30 minutes: an ``analysis of an argument.'' The argument that you will find on the test is
typically related to topics of general interest related to business or a variety of other subjects.
Manhattan Elite Prep - Verbal Concepts Guide | GMAT Elite Strategy Guide

19

The AWA section receives a score from 0-6, in increments of 0.5.

0 indicates incomprehensibility.

6 indicates a well-focused and clear essay.

The AWA score is NOT reflected in the combined verbal and quantitative overall score, but is still an important aspect
of your MBA application and should not be neglected. By focusing on clarity and precision rather than on saying
something complicated or brilliant, you will score higher on this portion of the GMAT.

The AWA section is designed to directly measure your ability to think critically through the complexities of an issue
and to communicate your ideas through substantiated reasoning.

In the Analysis of an Issue section, you will need to analyze the issue presented and explain your point of view on the
subject. There is no correct answer. Instead, you should consider various alternative perspectives. Use relevant reasons
and/or examples drawn from your experience, observations, daily reading, or general knowledge to develop your own
argument on the issue.

The Analysis of an Argument tests your ability to formulate an appropriate and constructive critique of a specific
conclusion based on a rigorous approach. In this section, you will need to analyze the supporting logics behind a given
argument and write a critique of that argument. Remember your task to examine and critique the given argument, not
to present your own views on the subject.

Consider the following when developing your essay:

The underlying debatable assumptions behind the argument

The alternative explanations or counter-examples might weaken the conclusion

The type of evidence could help strengthen or refute the argument

For the AWA section, we recommend that you make a consistent effort to:

Brush up on your typing skills.

Be careful not to make careless mistakes in spelling or grammar.

Make the reader aware of your essay structure. This can be done through formatting. Underlining, numbering or
making bullet points can reinforce your essays structure.

Others to be discussed in class.

1.4.7 Absolute vs. Partial Words

There is perhaps no other single aspect of the GMAT that will be more useful on more occasions, will cut the time and
effort youll need to devote to any individual question, and, most importantly help you boost your scores than learning
to recognize and question the use of Absolute and Partial words.

If you can find exceptions to these scenarios, then you can rule out those answer choices.
Manhattan Elite Prep - Verbal Concepts Guide | GMAT Elite Strategy Guide

20

Whats even more powerful is to realize that the identical idea applies equally to the Verbal and Quantitative portions
of the test.

Absolute Words
Absolute words on the GMAT are just that: absolute. They state ideas or relationships in terms of certainties, in black
and white terms, all or nothing, always or never: they leave no room for exceptions or nuance.

Lets begin making a list of Absolutes; keep this handy, as youll be adding to it as you encounter less and less obvious
attempts by the GMAT to slip one of these by you unnoticed.

Absolutes in the Affirmative:
Always All
Will Cures
Must Prevents
Can only Leads to
Causes Creates

While this list is by no means definitive, and well be adding to it as we encounter less obvious terms throughout the
test, one common theme is already evident: each of these words speaks in certainties. This is unlikely to be the case in
nature, and is usually a dead giveaway on the GMAT.

If an argument concludes by saying This will resolve the problem, or This causes undesirable results, we should
politely say Thank You, and keep firmly in mind that these conclusions are absolute; if you can imagine ANY
single circumstance under which this would NOT be true, then the entire conclusion if rendered
invalid. Rarely is the test more generous to us than this!

Absolutes in the Negative:
No Never
None No one
Cannot Will not
Does not Prevents
Avoids Rules out

Again, while this list is not exhaustive, you should be able to recognize that any argument that includes a term in this
category is built on a very, very shaky foundation: if we can think of ANY SINGLE case, which need not be
mentioned anywhere in the text, where one of these absolute statements could not stand up to the light of closer
scrutiny, then we have no choice but to discount the entire argument, case closed. Again, rarely does the test show us
this kind of generosity, once we train ourselves to recognize these terms. And it only gets better .

Please note: Just because a statement includes an absolute term (either for or against) does NOT mean that we
can automatically rule it out. Consider the simple statement, All human beings require food and air to live. This is
a pretty bold statement, which has to be true for EVERY one of the seven billion or so people on Earth. However, in
this case, after considering it for a moment, we can see that this extravagant statement is really so true. This is not
meant to be anything too terribly difficult to determine, since absolute statements ARE invalid considerably more often
than less stringent statements. It just means that, as is usually the case, that you cant automatically rule out an absolute
statement without at least briefly considering it first. Welcome to the real world, friends.

This leads us to our sister concept:

Manhattan Elite Prep - Verbal Concepts Guide | GMAT Elite Strategy Guide

21
Partial Words

Partial Words and their place on the GMAT, or How I learned not to get sued for breach of contract in one easy step!
Just as the name suggests, Partial Words are by no means definitive or absolute. In fact they span the entire range
from Words that tell us the next closest thing to absolutely nothing without actually telling us nothing to words that
may make a very strong and reasonable case for a particular argument, but which leave open the possibility that there
may be a relatively small number of exceptions.

Lets start our list with the weakest of partial words:
Some Sometimes
May Might
Can Could
A number of

The annoying trait that all of these words have in common is that, using plain language, we dont know if this situation
is common or rare, likely or unlikely. It could be true 1% of the time or 99%; without any more information, all
we can say is that the statement in this case is not absolute, it has exceptions (or at least one!)

The second type of Partial Words may actually end up giving us something we can use, such as:
Many Often
Commonly Frequently (same meaning)
Many people In many cases

While these words still dont give us any concrete numbers, our common sense tells us that these kinds of statements
are going to have more importance than some. They could be a fairly small or a relatively large part of the cases in
question, but its reasonably to work under the assumption that in many cases, for example, is probably somewhere in
the range of 20% to 40%. We dont need an actual percentage to know that this point is worth taking into account
before we decide anything.

Finally, we have the probable Partial Words, the strongest kind. These include:
Most Usually
Typically Generally
A majority Probably
Likely Can be expected to

With this final group of partial words, we still dont know the actual percentage of cases in which the conclusion is
true or not, the one thing we DO know is that its a probability; its more likely than 50% to be true. Such statements
are obviously very important to any argument; whereas some or might be tell us almost nothing, in contrast, terms
such as most consumers or is probably tell us something about more than half the population in question,
something we certainly would like to know.
Manhattan Elite Prep - Verbal Concepts Guide | GMAT Elite Strategy Guide

22
1.5 Frequently Asked Questions

1.5.1 Test Day FAQ

What is the registration process for taking the test?
Can I cancel my score, or have it not be reported?
How many times can I take the GMAT?
Are there any rules regarding how often I can take the GMAT in a certain number of days?
Is it common to retake the GMAT? If so, what kinds of results are common?
What should I do if I dont feel well on the day of the test?
Should I arrive early at the test center?
What should I bring to the test center on the day of my test?
What is the test center like? Will they let my family in with me?
What is the testing room environment like?
What should I wear? Is there a dress code?
What is the check-in procedure? Can you tell me more about the palm vein recognition system?
How long does the GMAT take? Are we given breaks?
Can we leave during the test?
Can I bring a snack? What are those items that I can access during breaks?
What are the rules for cell phone and PDA use?
What happens if there are extreme circumstances such as severe weather or computer malfunctions?
When I sit down at the computer, do I take the test right away?
What should I do if my computer does not work?
What if something happens to disrupt the test?
Can I use a testing aid such as a calculator?
Can the test administrator kick me out? What for?
Is there a plagiarism policy, regarding the writing section?
What happens after the test?
How do I send my scores to schools?
How can I tell if my GMAT score is high enough?

What is the registration process for taking the test?

Signing up for the GMAT is the easiest part of your process. Here is a web URL: www.mba.com/mba/TaketheGMAT.
Popular testing days fill up in advance, so if your schedule requires you to take the test on a certain day, be on top of it
and sign up early. You may register up to 6 months in advance. If you register online or by phone (an additional fee of
$10 may apply), you may register as late as 24 hours prior to your test date as long as there is availability. To allow
sufficient reporting time, please also choose a test date at least 21 days prior to your application deadline. You need to
pay the fee with a major credit card.

Can I cancel my score, or have it not be reported?

You can cancel your score, but you must do so before you see it. Once you see it, it must be reported; you do not have
the option of withholding it. We do not recommend canceling a score unless you were tested under adverse
circumstances, like illness. If you do poorly and still report your score, it is not the end of the world. An improvement
Manhattan Elite Prep - Verbal Concepts Guide | GMAT Elite Strategy Guide

23
between two test scores may be highly regarded by your school of choice. If you cancel, the cancellation is still
reported to the schools.

How many times can I take the GMAT?

There is technically no limit but we do not recommend taking the test more than three times. Taking the test many
times may look bad. Plan on taking the test twice and reporting both scores (unless of course you do very well the first
time). The admissions committee will take the best score of all your tests but will take a look at the history of the tests
as well.

Are there any rules regarding how often I can take the GMAT in a certain number of days?

You can take the GMAT five times every year (12-month period). Within that year, you must wait 31 days between
tests, regardless of score cancellations.

Is it common to retake the GMAT? If so, what kinds of results are common?

It is less common than you might think. Studies show that approximately 21% of the exams are taken by those who are
retesting. Of those re-testers, it is uncommon that they take the test more than 3 times in a given year. Regarding
results, the average increase between tests is 30 points on the total score. It is not unheard of that scores can go down
with repeat testing. Your increase will depend on how much preparation you do in between. If you prepare well
enough, odds are you wont be compelled to retest!

What should I do if I dont feel well on the day of the test?

If you dont feel well, you should not take the test. You should be healthy and prepared when you take the GMAT. It
will be an ordeal to sit through a 4-hour test while you are not at your best. You may end up canceling the score
anyway. So do not get hung up on the rescheduling fee (US$50 if you reschedule before 7 calendar days of the test
date or you will lose the entire US$250 test fee). Your bad score might become an anchoring point later on hindering
your progress and hurting your self-confidence. Meantime, a bad score gets reported to schools as all your scores in the
last five years will be shown.

However, if you are sure that you can achieve a high score based on your prior practice tests, you should take the test
even when you dont feel well. That way you dont waste the test fee and have a chance to experience the real test
while having the option to cancel it at the end.

Should I arrive early at the test center?

Yes. Make sure you find out in advance where the test center is, and get there at least 30 minutes early because there is
a check-in procedure. If you are more than 15 minutes late, the test administrators may not let you take your test and
your US$250 test fee could be forfeited.

What should I bring to the test center on the day of my test?

You do not need to bring much to the test center. In fact, you will not be allowed to bring any of your personal
belongings such as cell phones, backpacks, purses, or handbags into the testing room. If you do bring your belongings
with you, you will be required to put them in a storage area such as a small locker. So just bring the essential items
(including clothes) you need before, during and after the test in a bag with flexible surface. The test administrator will
provide you with everything you need to take the test, including a pencil and scratch paper/erasable booklets.

What is the test center like? Will they let my family in with me?
Manhattan Elite Prep - Verbal Concepts Guide | GMAT Elite Strategy Guide

24

The test centers are typically very small. Depending on their locations and relationships with Pearson Vue, they might
be owned by Pearson VUE or just an affiliated one in the network. If they are the latter, they may administer other tests
for non-Pearson VUE entities. Therefore the condition of the test center and your test center experience may vary.
There is a waiting area. Depending on the number of staff members on duty at the test center and the number of test
takers waiting, your wait time may vary. Feel free to relax. If you want to continue to read notes or books while
waiting, lock your belongings away in the locker first except for a book or some notes. When you get called, you can
quickly go to your locker and lock the last item before proceeding. But please be quick, otherwise the staff may
become impatient.

Your family or friends will not be permitted to wait for you at the test center while you are taking your test. They will
also not be able to contact you while you are being tested.

You will take your GMAT at an individual computer workstation next to many other computer workstations in a
testing room. Your computer workstation is similar to a basic small office cubicle. Please note that there will be
audio/visual monitoring during the test.

What should I wear? Is there a dress code?

There is no dress code. You should dress comfortably and possibly bring extra layers in case the room gets cold.

What is the check-in procedure? Can you tell me more about the palm vein recognition system?

You will need to show the test administrator valid ID. Please prepare two to three valid IDs such as a passport, drivers
license, or a major credit card with your photo. That way in case one expires or somehow cannot be found at the test
center, you are still good to go.

The administrator will also take your photograph, signature, and take your palm vein digitally. You will be asked to
agree to the GMAT Examination Testing Rules & Agreement. When you sign for the paperwork, please slow down
and make your signature very legible and correspond to the full name on your ID. Otherwise you may be asked to re-
sign your name multiple times which might affect your mood for the test later on. If you refuse to comply with any of
these procedures, you may be asked to leave and forfeit your fee.

Palm vein recognition works by scanning the veins inside of your hand and creating a digital template that represents
your vein pattern, which is unique to each individualeven identical twins have different patterns. You just need to
hold your hand a few inches above a sensor. The system is highly accurate, widely applicable, less intrusive and more
culturally acceptable than some other identity verification technologies, such as digital fingerprinting. In addition, it
does not cause any health concern. Your palm vein pattern will be matched before and after you start an exam and
return to the testing room after a break. Your palm vein patterns will also be compared with those of other test takers to
allow Pearson VUE to find people who may have tested under multiple names or identities. Your vein pattern
template is stored separately from other information about you in the system. Your vein pattern template is not
disclosed outside of GMAC and Pearson VUE, except as required to prevent illegal activity and detect fraud. While
schools you have selected to receive your GMAT score may elect to receive a copy of your digital photograph with
your test score, they do not receive your signature or your palm vein pattern template.


How long does the GMAT take? Are we given breaks?

The test takes around 4 hours. You are given two 5-minute breaks at scheduled intervals. It is important to not take
more than 5 minutes during these breaks because you will be docked the amount of time you go over.

Manhattan Elite Prep - Verbal Concepts Guide | GMAT Elite Strategy Guide

25
Can we leave during the test?

Not besides the scheduled breaks. If there is an emergency, raise your hand and the test administrator should let you
leave; however the timer on your test will not stop. Be careful about leaving, because if you are gone too long or leave
too many times, your test administrator may report you to Pearson VUE or GMAC. Note that you will need to place
your palm above a palm vein sensor at re-entry.

Can I bring a snack? What are those items that I can access during breaks?

Yes, but not into the testing room. You can only eat, drink, or use tobacco or access comfort items such as tissues or
cough drops during the scheduled breaks. If you need to smoke or go outside for any reason, be sure to alert the test
administrator. Note that rules about leaving vary from center to center, so you should clearly understand those rules
before making any decisions in order to avoid any problems. We recommend you bring some chocolates, power bars
and water for two short eight-minute breaks in order to maintain your momentum through the test.

Be sure to remember not to access any electronic devices or study material during breaks. Your scores may likely be
withheld or cancelled, the incident reported to schools by the test administers and noted on Official Score Reports for
five years.

What are the rules for cell phone and PDA use?

You cannot use your cell phone or PDA during the test or on your break. We recommend you turn off any electronic
devices stored in the locker. Please do not check your voicemail or emails during your break. You will be violating the
test center rules and may be asked to leave immediately.

What happens if there are extreme circumstances such as severe weather or computer malfunctions?

Usually, the start of the test will be delayed or will be rescheduled. If such action is not possible, you can choose either
a full refund or to sign up for a different test date free of charge.

When I sit down at the computer, do I take the test right away?

You will have to begin your test as soon as possible, but first you will have to agree to the GMAT Nondisclosure
Agreement and General Terms of Use statement including that you will not use any non-permitted devices. You must
agree to this statement, or risk not being allowed to complete the test and forfeiting your test fee.

What should I do if my computer does not work?

You should raise your hand and tell the test administrator. You should not try and fix it yourself.

What if something happens to disrupt the test?

Based on the policies listed on the websites of GMAC, Pearson VUE and ACT, if anything happens during the test
that serves as an interruption or causes a mistiming, or if anything is out of the ordinary or deviates from normal
testing procedure, GMAC and/or Pearson VUE will conduct an examination of the situation. One of those
organizations will then decide if they should pursue disciplinary action including score cancellation. If any
organization decides to take any action against an examinee or on behalf of one, the affected parties will have the
option of A) take their test again at no additional charge, or receive a refund UNLESS that person is found to be the
cause or source of the disruption. If you should opt to take the test again, you must take it in its entirety.

Can I use a testing aid such as a calculator?
Manhattan Elite Prep - Verbal Concepts Guide | GMAT Elite Strategy Guide

26

No, you cannot use a calculator or any other type of aid including laptops, PDAs, translators, dictionaries etc.

However, for the Integrated Reasoning section, there is an online calculator. Please note that such online calculator is
not available for the Quantitative section.

Can the test administrator kick me out and what for?

Yes. There are many reasons for dismissal. These include providing false information or not providing it at all,
bringing a test aid, not complying with an administrators reasonable requests, helping another test taker or letting
them help you, tampering with the operation of the computer, and not following any rules or procedures.

Is there a plagiarism policy, regarding the writing section?

You must write one essay for the Analytical Writing Section of the GMAT. If there is evidence of plagiarism, your
scores can and will be cancelled. Please note that your essay becomes the property of GMAC.

What happens after the test?

After the test you will answer some questions about yourself and your plans for graduate school, and whether you
would like to take part in various surveys or receive further information. Your answer choices to these questions may
reflect information that you previously provided, earlier in the registration process.

How do I send my scores to schools?

There are a few options for sending your scores. After you take the test, you will have the option of selecting 5
programs to send your scores to at no extra charge. Be careful, because this action cannot be undone and also be sure
to select the correct program.

Later, you will have the chance to choose more schools to send your scores to. You will need to order an Additional
Score Report (ASR). You can pay by credit card or check. Each additional school costs $28 USD.

How can I tell if my GMAT score is high enough?

It depends what you mean by high enough. Different schools have different expectations as to what GMAT scores
admitted students should have. Some schools have a minimum requirement you will need to research this on a
program-by-program basis, including research on what the range of scores was for the last entering class. Your GMAT
score is only one third of your application, so let your score be a rough guide to where you apply but do not limit
yourself. Also, if you are unhappy with your score, it is fairly common to take the GMAT twice. If you take it twice,
be sure to prepare thoroughly in order to achieve a higher score.


1.5.2 General FAQ

What is the GMAT?
Does the GMAT have more or less influence on your application than other things, like GPA, work
experience, recommendations and interviews?
Is there any way I can apply to business school without reporting a GMAT score?
Can you explain the different parts of the test in more detail?
How is the GMAT scored?
Manhattan Elite Prep - Verbal Concepts Guide | GMAT Elite Strategy Guide

27
Do I need any special computer skills to take the GMAT on a computer?
Are there accommodations for disabled test takers?
What level of math does the GMAT cover?
What is the content of the reading passages? Should I be prepared to read about business?
What is the advantage of joining a classroom prep program like Manhattan Elite Prep as opposed to just
working with a study book?
Should I prepare for my GMAT with books, or with programs that simulate the CAT?
How are the essays scored?
If I take the GMAT a number of times, will my scores vary?
How does the CAT work, as in, how does the test adapt to the individual user?
Should I respond incorrectly to the harder questions in the beginning on purpose, so that the questions I get
later I can answer easily?
Are there other factors involved in the scoring?
In this system, do you advise against random guessing?
Why are there un-scored questions?
Are there any strategies unique to the CAT?
How does the CAT produce a reliable measurement with so few questions?
Can you discuss the CAT algorithm?
What are some differences between the CAT and the old style test?
Some of the Integrated Reasoning questions require multiple responses. Do I get partial credit if I get just
some but not all of the parts correct?



What is the GMAT?

The GMAT is the standardized test that you take when applying to business school. It lasts between 3.5 and 4 hours.
The format is as follows: First you write one essay that last a half hour, second you spend another 30 minutes on
Integrated Reasoning questions, after which you have a 5 minute break. Second, you spend 75 minutes on the
quantitative section, which has 37 questions, and you take another 5-minute break. Last is the 75-minute verbal
section, which is 41-questions long.

The GMAT, along with your undergraduate grade point average (GPA) and your work experience are the three things
that admissions committees look at when considering you for admission. The GMAT is important and should be taken
seriously, especially by those who are not happy with their undergrad GPA or have very little work experience.

Does the GMAT have more or less influence on your application than other things, like GPA, work experience,
recommendations and interviews?

Your GMAT score is very important to the application process, but what is most important is determined per
individual. A high score can serve to undermine a lower GPA or less job experience, or set you apart from those with a
very high GPA and outstanding recommendations, but who are poor test takers. Conversely, if you have a very high
GMAT score, a good GPA and job experience can set you apart from others with high GMAT scores.

Is there any way I can apply to business school without reporting a GMAT score?

No, unless the schools or the programs (such as some executive MBA programs) dont require it or accept GRE scores.
Otherwise, you must report a GMAT score to be considered for admissions at any accredited MBA program.

Can you explain the different parts of the test in more detail?
Manhattan Elite Prep - Verbal Concepts Guide | GMAT Elite Strategy Guide

28
Yes. The four parts are the Analytical Writing Assessment, Integrated Reasoning, Quantitative section, and Verbal
section. Here is some more information on each:

Analytical Writing Assessment
The exam begins with one essay. You are given 30 minutes. The essay is intended to demonstrate the writing skills as
required in an Analysis of an Argument.

Integrated Reasoning
You are given 30 minutes for a total of 12 questions. It tests your ability to understand and evaluate information based
on graphic numeric and verbal sources. The problems includes text passages, tables, graphs and other visual
information and requires either of or a combination of quantitative and verbal reasoning skills.

Quantitative Section
After your 5-minute break, you move on to the quantitative section of the exam. There are 37 questions in two
categories: data sufficiency and problem solving. You will have 75 minutes to complete this section.

Verbal Section
After your second 5-minute break, you move on the verbal section. There are 41 questions of 3 types: Reading
Comprehension, Critical Reasoning, and Sentence Correction. Again, you will have 75 minutes to complete this
section.

How is the GMAT scored?

The GMAT is scored between 200 and 800, with 580 being close to the average score. In recent years, there have been
more test takers yielding higher scores however. Breaking 650 could get you into many schools, but some top schools
require above a 700.

In addition to your overall score, you receive a sub score for the verbal and quantitative sections, and a percentile. The
percentile means that you did better than that percent (the one they gave you) of the testing population in the last three
years from the date of your score is reported. The percentiles for quantitative and verbal refer to different scores,
because more people do well in quantitative on the GMAT. This goes to show that in your GMAT prep, you should be
thorough and cover both sections. Cover whichever you are weaker in more.

Along with your scores, your essay will be sent to your schools of choice.

Do I need any special computer skills to take the GMAT on a computer?

No you dont. But the better you are with computers, the more comfortable you will feel with taking a computer test,
making it more likely for you to receive a higher score. At the least, you need to know how to operate a computer
generally, as well as type and use a computer mouse. There is a tutorial before you start the test that is un-timed. The
tutorial will teach you how to answer questions and proceed through the test. Since there is one timed essay, you
should be comfortable with typing at a reasonable speed.

Are there accommodations for disabled test takers?

Yes. There should be an accommodation to suit any disabled test taker, whatever the disability may be. These
accommodations include but are not limited to the following: More time, more breaks or extended breaks, allowance
of medical devices like insulin pumps, a different mouse, a reader, sign language interpreter, and a larger font on the
computer monitor. Some allowances or comfort require special permission through the GMAT Test Accommodations
Request Form, and some do not. Those that do not are: glasses, hearing aids, a support pillow, a neck brace, and again
an insulin pump. The insulin pump must be attached to the body, or else it will need special permission.
Manhattan Elite Prep - Verbal Concepts Guide | GMAT Elite Strategy Guide

29

What level of math does the GMAT cover?

The GMAT covers through high school math. You do not need advanced math knowledge to be successful on the
quantitative section of the GMAT, but you need to be able to apply your math knowledge and use your skills in diverse
ways. The best thing to do is to prepare rigorously with GMAT experts and use any tools at your disposal to practice
your math skills.

What is the content of the reading passages? Should I be prepared to read about business?

The reading passages can cover just about anything. This part of the test is not testing you on specific subject matter,
but on your comprehension of the subject matter. The content, in that sense, does not matter.

However, the more knowledgeable you are about the tested topics in Reading Comprehension, the more likely you can
speed through the reading with more accurate answers.

You may see as many as 4 passages in the Reading Comprehension section ranging up to 400 words in each passage,
followed by 3 or 4 interpretive, applied, and inferential questions. The topics are typically related to social sciences
such as politics and history, physical or biological sciences such as geology and astronomy, business-related areas such
as marketing, economics and human resource management, along with other advanced subjects.
What is the advantage of joining a classroom prep program like Manhattan Elite Prep as opposed to just working
with a study book?

Joining a prep course is a good option because you are learning from an instructor. You are able to ask the instructor
questions. Asking questions is an important part of learning a part that is lost when you learn from a book. At
Manhattan Elite Prep our instructors are both experienced educators and very high scorers on the GMAT. Also, being
in a classroom setting can be conducive to good study habits. You also have the option of reviewing material and
studying with a fellow classmate.

There are many good reasons to join a prep course, not the least of which is that you will have an instructor who has
the knowledge to answer most of your GMAT questions. At Manhattan Elite Prep, our instructors have been teaching
students how to prepare for the GMAT exam for years, and they themselves have scored in the 99th percentile. To
involve yourself in a classroom atmosphere also opens you to questions that your fellow students might have,
questions you may not have thought of yourself. Importantly also, the classroom environment will make you a more
diligent studier. Coming to class will keep you on top of your preparation and will promote good study habits.

Should I prepare for my GMAT with books, or with programs that simulate the CAT?

Our advice is to do both. Books will help with reviewing grammar rules, vocabulary, and math skills but you will need
to practice putting yourself in a testing situation and you will want this to closely reflect the real GMAT, which is
where a computer program will come in. You can determine the pace at which you should take the real test, get used to
the computer atmosphere, practice your typing, and learn how to gauge your performance. If you take a practice CAT a
few times, you will have a better idea of how your scores will come out and you can make a more educated decision,
when the real GMAT comes, of whether to keep the score or not.

This is not to say that paper based practice tests are not worth your while. On the contrary they are worth it: any
amount of GMAT practice is worth your time. Also, be sure to research the materials you plan on using. Not all
software recreates the GMAT as well as others.

How are the essays scored?

Manhattan Elite Prep - Verbal Concepts Guide | GMAT Elite Strategy Guide

30
They are scored between 1.0 and 6.0 in half point increments. This score does not factor into your GMAT total score.
While your AWA score is reported to schools, it is not as important as your GMAT total score. However, schools may
compare your AWA writing to your application essays to ensure that your essays are reflective of your own writing
abilities.

If I take the GMAT a number of times, will my scores vary?

There is a degree of error with the GMAT as with all standardized tests. The standard error of difference for the total
GMAT score is about 41, according to Graduate Management Admissions Council. This means that the difference
between the total GMAT scores actually received by two test takers could be within 41 points above or below the
difference between the test takers' scores of true ability. The standard error of difference for the Verbal scaled score is
3.9, and for the Quantitative scaled score 4.3.

Research also indicates that a test-taker will most likely earn a Total score within about 30 points of a score of true
ability. Your Verbal and Quantitative scaled scores are probably within about 2.9 points of your true scores.

It is important that not everyones experience will be in accord with this purely mathematical understanding of the test.
There are situations where a person can score dramatically lower (probably not higher) than what there score should
be. For example if you are very sick, or your beloved dog dies, you could score significantly lower than 29 points
below your previous score.

GMAT scores are a relatively reliable predictor of academic performance in the first year of a business school
program. Studies have shown that the median correlation between GMAT scores and first-year grades was 0.41
(perfect correlation is 1.0). Comparing 0.41 to the median correlation of 0.28 between undergraduate grade point
average and first-year grades, you can conclude that business schools do have a strong incentive to see good GMAT
scores from applicants. Because there is a degree of error, we all should exercise caution in comparing two scores.
That is why other parts of your business school application are also as crucial to your admission.

How does the CAT work, as in, how does the test adapt to the individual user?

The multiple choice sections of the test are able to gauge how you respond to questions of various difficulties, and to
give you questions based on that information.

In each question type, for example, sentence correction you will get a question of average difficulty first. If you answer
this question correctly, your next questions will be more difficult. The more questions you answer correctly, the more
difficult they will become. If you answer incorrectly to the first, average-difficulty question, your second question will
be less difficult. This has two consequences: The first is that as you take the test, you will get fewer questions that are
very easy for you, and fewer that are too hard. Also, no other test taker will see the same question combinations. In
those ways, the CAT is able to better gauge your performance.

An ideal item pool for a computer adaptive test would be one with a large number of highly discriminating items well
distributed at each ability level. The information functions for these items would appear as a series of peaked
distributions across all levels of ability estimate.

Please note that the Integrated Reasoning section is not computer adaptive. It is timed and you cannot skip any
questions or go back and change your answers once you have submitted them.

Should I respond incorrectly to the harder questions in the beginning on purpose, so that the questions I get later I
can answer easily?

Manhattan Elite Prep - Verbal Concepts Guide | GMAT Elite Strategy Guide

31
This may seem like a valid strategy, but you should not do it. You are not only scored on the number questions you
answer correctly, but also on the level of difficulty of those questions. So, answering 10 hard questions correctly
counts for more points than 10 easier questions.

Are there other factors involved in the scoring?

Yes there is one more. If you can answer a number of questions correctly that cover a range of question types, your
score will improve. For example, if you can answer questions on algebra, geometry, and statistics in both data
sufficiency and problem solving correctly, you will score higher than doing so in only one of those areas. This third
factor however is less important in determining your score.

The logic behind having the third factor is simple: A business manager must be well rounded. He or she cannot just be
an expert mathematician or a powerful analytical thinker. It is more important that the test taker prove that he or she is
able to do both at a high level.

In this system, do you advise against random guessing?
Yes, we do advise against random guessing, for the most part. The only benefit of a random guess is that it will save
you time on a question that leaves you particularly stumped. However with the odds at 1 in 5, the benefit is not worth
the risk. Since the odds are that you will get it wrong, your next question or questions will be easier, and you will be
losing the opportunity to answer difficult questions, thus losing the opportunity to max out your score. Also, the
GMAT only gives the test taker 37 Quantitative and 41 Verbal questions, 10 of which on each are un-scored. It follows
that you need to do you best on each question because you dont have that many chances.

The problem with this advice is that there is a penalty for leaving a section blank. This penalty is worse than an
incorrect answer. That being said, if you cannot figure it out, you can probably narrow it down to 3 or 4 options. Make
educated guesses, not random ones, UNLESS you are scrambling for time at the end of the exam. That is the only
situation in which completely random guessing would be suitable.

It is even more important to not guess in the beginning of the test. These first few questions more or less determine the
difficulty level of the rest of the test. It will be nearly impossible to convince the CAT to change the difficulty level
in the middle. If you guess incorrectly on one or more of the first few questions, you will be stuck in a low-difficulty
testing situation, and will not be able to make up points later. It is unlikely that after establishing a low difficulty level,
the CAT will randomly introduce more difficult questions.

Why are there un-scored questions?

There are constantly new questions being added to the GMAT. A new question will be in the bank of un-scored
questions so the testers can determine the difficulty level, before making it an officially scored question. You will not
be able to tell the difference between a scored and an unscored question, we do not recommend that you try.

Are there any strategies unique to the CAT?

Yes. You should be careful and take your time in the beginning of the multiple-choice sections, and whenever you see
the first question of a given type. Firstly, the beginning of the sections determines, to a large extent, the level of
difficulty in the remaining questions. Secondly, since you want to answer the widest variety of questions as possible
correctly, when you get to a new question type it is important that you do your best to get it right; meaning read
carefully and double check your answers. Please note that with any kind of testing, it is important to take your time,
but you need to budget your time such that you can complete the whole test without feeling rushed. Also, while it is
always good to double check, it can be problematic to change your answers from what you thought first. Multiple-
choice tests tend to rely on intuition, and changing your answer could result in a wrong answer.

Manhattan Elite Prep - Verbal Concepts Guide | GMAT Elite Strategy Guide

32
How does the CAT produce a reliable measurement with so few questions?

An adaptive test, whether computer or paper based, yields a wider range of scores than a non-adaptive test. A wider
range of scores can serve as a more accurate way of comparing test takers. That CAT yields a wider range of scores
because the easier questions count for less. We know that someone who answers 10 easy questions correctly will get
fewer points that a person who answers 10 hard questions correctly. It follows that the first person may have less
cognitive ability, and in turn have a much lower score.

Can you discuss the CAT algorithm?

Yes. The algorithm is an iterative process.

Step 1: Given the currently estimated ability level of a test-taker at a given point (usually the first question is started at
mid ability level), the program evaluates all the items that have not yet been administered to determine which will be
the best one to administer next.

In this approach, the best next item would be the one that provides the most information about the test-taker.
Typically difficulty level of an item is the most important parameter. However, in order to be able to clearly
discriminate the ability among individual test-takers, the test-maker also incorporates other factors in the item selection
process on a particular exam. They include different question types (data sufficiency vs. problem solving; critical
reasoning vs. sentence correction), content (e.g., algebra, ratios, combinatorics, topic and inference questions for the
same reading comprehension passage, etc.), and exposure (i.e., the number of times the question has been seen by
other test takers already during a given period).

Demonstrating to the CAT that you can handle a variety of substantive areas in all question formats will increase your
GMAT score. The greater the variance among your ability in different tested topics, the lower your score. In other
words, the GMAT rewards generaliststest takers who demonstrate a broad spectrum of competencies. This approach
does make sense as in a business world, being well-rounded and knowledgeable can be positively correlated to a
managers decision-making skills and managerial ability in general.

Step 2: The "best" next item is administered and the test-taker answers.

Step 3: The program computes a new ability estimate based on the answers to all of the previous items.

Steps 1 through 3 are repeated until a stopping criterion is satisfied.

What are some differences between the CAT and the old style test?

The Cat is a more efficient test taking process. The old paper and pencil test had 61 questions in each section, all of
varying difficulty. The test taker would, in affect, waste time answer questions that were too easy for them, and
stumbling around difficult questions that they would probably get mostly wrong. The CAT is shorter, and avoids
questions that are either gimmees, or much too hard.

Some of the Integrated Reasoning questions require multiple responses. Do I get partial credit if I get just some but
not all of the parts correct?

No. You must get all parts of the question correct to receive any credit.

Manhattan Elite Prep - Verbal Concepts Guide | GMAT Elite Strategy Guide

33
1.5.3 Course FAQ

Why should I prepare for the GMAT?
What is the difference between preparing with books/CDs/online material and participating in your course?
I have studied mathematics/physics. Should I still take the course?

Why should I prepare for the GMAT?

Studies indicate that applicants who prepare for the GMAT score substantially higher than those who don't. In addition
to the admissions process, GMAT scores are also considered in the process of awarding financial assistance to
students. A good GMAT score can save you thousands of dollars in tuition.

What is the difference between preparing with books/CDs/online material and participating in your course?

We are frequently asked about the advantages of preparing with us rather than with books, CDs and online resources,
which are available in countless numbers and great volume. We readily admit that you can prepare well with such
materials. We believe that everyone who takes our course will receive a higher score within a shorter preparation time
period than if they study exclusively with books, CDs and online resources.

In sum, preparing for the GMAT with our course has the following major advantages:

We offer tips & tricks, not readily available in common test preparation literature.

No book or CD or online material will answer your questions, walk you through the most efficient means of
getting the right answer, or grade your analytical writing assessments upon your request.

Visualization and discussion in a seminar environment will enable you to recognize complex structures better
than learning the same material in a non-interactive way.

The priceless opportunity to talk to actual individuals who have been admitted to an Ivy League Universities
offers you the insights you can gain nowhere else.

You will be more motivated and have more fun during the preparation!


I have studied mathematics/physics. Should I still take the course?

Although some of the standard quantitative techniques we teach might be repetitive for you, many parts of the course
focus on more advanced and lesser-known techniques to increase the accuracy of your answers, in particular for the
Data Sufficiency section. In addition, our course contains many aspects commonly neglected in the studies of
mathematics at universities.

You might also like